LSAT and Law School Admissions Forum

Get expert LSAT preparation and law school admissions advice from PowerScore Test Preparation.

 Administrator
PowerScore Staff
  • PowerScore Staff
  • Posts: 8917
  • Joined: Feb 02, 2011
|
#23448
Complete Question Explanation

Flaw in the Reasoning-CE, #%. The correct answer choice is (D)

This is a fairly recurrent theme in the logical reasoning section of the LSAT: based on a correlation, the author draws a flawed causal conclusion:
  • Premise: ..... The most heavily patrolled neighborhoods are those with the most crime.

    Conclusion: ..... Police presence does not decrease crime.
This reasoning is quite questionable—the author seems to want to imply that police presence actually increases crime:
  • Cause ..... ..... ..... Effect
    Police presence ..... crime rate increase
Another, perhaps more reasonable explanation for the referenced statistic would be that the police are most likely to patrol where they are most needed—neighborhoods with relatively high crime rates.

Answer choice (A): There is absolutely no appeal to emotions in the stimulus, so this answer choice is incorrect.

Answer choice (B): There is no reason to consider the possibility that criminals might move from neighborhood to neighborhood, so this cannot be the flaw in the stimulus' reasoning.

Answer choice (C): There is no basis for the assertion that the author's argument is based on too small a sample of data—there is no way to assess the specifics of the "statistical evidence" that the author discusses.

Answer choice (D): This is the correct answer choice, restating our prephrased answer in the discussion above. The author does not consider the possibility that police presence is placed where it is most needed.

Answer choice (E): The author does not presume that there is some other system of allocation that would provide a stronger deterrent, so this answer is incorrect.
User avatar
 desiboy96
  • Posts: 45
  • Joined: Jan 20, 2021
|
#83568
Hello, so I did this question and chose D because all of the other one's seemed incorrect. Just to clarify is D correct because it provides an alternative conclusion based on the statistics cited correct?

I'm a bit confused because while I did recognize that a causal flaw, or lack thereof in this case, is being committed here, I was thinking in my head that the correct answer will refute the conclusion by using the statistics to show how police presence does in fact decrease crime.

Sorry if this sounds like a silly question. I'm just going over LR questions more now so that I can catch little mistakes/ bad habits that I'm making so I can break them.
User avatar
 KelseyWoods
PowerScore Staff
  • PowerScore Staff
  • Posts: 1079
  • Joined: Jun 26, 2013
|
#83590
Hi desiboy96!

Careful with your prephrase here. You state that you were looking for an answer choice that would "refute the conclusion by using the statistics to show how police presence does in fact decrease crime." This is a Flaw in the Reasoning question. In a Flaw question, we are not trying to refute a conclusion. All we are trying to do is identify the flaw. A flaw just means that the premises given do not prove the conclusion as stated. So all we're looking for is an answer choice that explains why the premises do not prove the conclusion. As you identified, this is a causal conclusion and those are inherently flawed. The reason that causal conclusions are inherently flawed is because there are often other explanations for any posited causal relationships. Let's break down the premise and conclusion again:

Premise: Neighborhoods with most police have most crime.
Conclusion: Police presence does not decrease crime.

Here, the author is essentially taking a premise that provides a correlation and making a causal conclusion. One of the main reasons why we can't draw causal conclusions from correlations is because even if the two events are causally related, we do not know the direction of that causal relationship. It doesn't have to be that the greater police presence is causing more crime; it could be that more crimes are causing the greater police presence.

So when we're identifying the flaw, we just want to think about why the premises do not fully prove the conclusion. In this case, the correlation data does not fully prove the causal relationship because there is another valid interpretation of that data that the author has not considered. Answer choice (D) describes that flaw and is, thus, correct.

Hope this helps!

Best,
Kelsey

Get the most out of your LSAT Prep Plus subscription.

Analyze and track your performance with our Testing and Analytics Package.